You are on page 1of 5

Homework 3

1. If the inverse T −1 of a closed linear operator exists, show that T −1 is a closed linear
operator.

Solution: Assuming that the inverse of T were defined, then we will have to have that
D(T −1 ) = Ran(T ). Suppose that {un } ∈ D(T −1 ) is a sequence such that un → u and
T −1 un → x. We need so that that u ∈ D(T −1 ) and T −1 u = x. Then by the simple
criteria, we would have that T −1 is closed.

Since un ∈ D(T −1 ) we have that un = T xn , where xn ∈ D(T ). Also, note we have that
T −1 un = xn → x. So, we have a sequence of vectors {xn } ∈ D(T ) with xn → x and
T xn → u. Since T is closed, we have that x ∈ D(T ) and T x = u. But, this last equality
implies that u ∈ D(T −1 ) and that x = T −1 u, which is what we wanted.

2. Let T be a closed linear operator. If two sequences {xn } and {x̃n } in the domain of T
both converge to the same limit x, and if {T xn } and {T x̃n } both converge, show that {T xn }
and {T x̃n } both have the same limit.

Solution: Suppose that {xn }, {x̃n } ∈ D(T ) are such that xn → x and x̃n → x. Let
T xn → y and T x̃n → ỹ. We need to show that y = ỹ.

Since T is closed, then we know that for xn → x and T xn → y implies that x ∈ D(T )
and T x = y. Similarly, we have that x̃n → x and T x̃n → ỹ implies that x ∈ D(T ) and
T x = ỹ. But, this then gives y = T x = ỹ, and we they have the same limit as claimed.

3. Let X and Y be Banach spaces and T : X → Y an injective bounded linear operator.


Show that T −1 : Ran T → X is bounded if and only if Ran T is closed in Y .

Solution: First suppose that Ran T is closed in Y . By problem 1 we know that T −1 is


closed and D(T −1 ) = Ran(T ). Then by the Closed Graph Theorem, we have that T −1
is a bounded linear operator from Ran T → X.

Now, suppose that T −1 : Ran T → X is bounded, i.e., kT −1 ykX ≤ c kykY . The goal
is to show that y ∈ Ran T . Let y ∈ Ran T , and then there exists xn ∈ X such that
T xn := un → y. We then have that the sequence {xn } is Cauchy since

kxn − xm kX = T −1 T xn − T −1 T xm X = T −1 (un − um ) X ≤ c kun − um kY


and since un → y this gives that the claim. Since X is complete, let x denote the limit
of the sequence {xn }. Then we will show that y = T x ∈ Ran T , and so the range of T is
closed. But, note that

ky − T xkY = ky − un + un − T xkY
≤ ky − un kY + kun − T xkY
≤ ky − un kY + kT k kxn − xkX .

Here we have used that T is bounded. But, the expression on the right hand side can be
made arbitrarily small, and so y = T x as desired.

4. Let T : X → Y be a bounded linear operator, where X and Y are Banach spaces. If T is


bijective, show that there are positive real numbers a and b such that

a kxkX ≤ kT xkY ≤ b kxkX ∀x ∈ X.

Solution: Since T is a bounded linear operator, then we immediately have that

kT xkY ≤ kT k kxkX ∀x ∈ X.

However, since T is bijective and bounded we have that T −1 is bounded as well. Note
that T −1 : Y → X, so for any y ∈ Y we have that
−1
T y ≤ T −1 kyk .

X Y

Apply this when y = T x, and we then have

kxkX ≤ T −1 kT xkY .

Combining inequalities we have that

a kxkX ≤ kT xkY ≤ b kxkX ∀x ∈ X


−1
with a = kT −1 k and b = kT k.

5. Let X1 = (X, k·k1 ) and X2 = (X, k·k2 ) be Banach spaces. If there is a constant c such
that kxk1 ≤ c kxk2 for all x ∈ X, show that there is a constant C such that kxk2 ≤ C kxk1
for all x ∈ X.

Solution: Consider the map I : X2 → X1 given by Ix = x. Then I is clearly linear


from X2 → X1 , and by the hypotheses, we have

kIxk1 ≤ c kxk2 ∀x ∈ X

so I is bounded. But, we also have that I is bijective, and so by the Bounded Inverse
Theorem, we have that I −1 : X1 → X2 is a bounded linear operator as well, i.e., there
exists a constant C such that
−1
I x ≤ C kxk ∀x ∈ X.
2 1

However, I −1 x = x and so
kxk2 ≤ C kxk1 ∀x ∈ X.

6. Let X be the normed space whose points are sequences of complex numbers x = {ξj }
with only finitely many non-zero terms and norm defined by kxk = supj |ξj |. Let T : X → X
be defined by    
1 1 1
T x = ξ1 , ξ2 , ξ3 , . . . = ξj .
2 3 j
Show that T is linear and bounded, but T −1 is unbounded. Does this contradict the Bounded
Inverse Theorem?

Solution: It is pretty obvious that T is linear. Since x = {ξn } and y = {ηn }, then we
have that      
1 1 1
T (x + y) = (ξj + ηj ) = ξj + ηj = T x + T y.
j j j
The case of constants λ, is just as easy to verify, T λx = λT x. It is also very easy to see
that T is bounded, since we have for any x ∈ X that

|ξj | 1
kT xkX = sup ≤ kxkX sup ≤ kxkX .
j≥1 j j≥1 j

We now show that T −1 is not bounded. First, note that T −1 is given by

T −1 x = {jξj } .

Now let x = {ξj } where ξj = 0 if j 6= n and 1 if n = j. Then we have that kxkX = 1


and T −1 x = {jξj } = (0, . . . , 0, n, 0, . . . , ) where the value 1 occurs in the nth coordinate.
This then implies that kT −1 xkX = n. But, this is enough to show that the operator
is not bounded. This doesn’t contradict the Bounded Inverse Theorem since X is not
complete.

7. Give an example to show that boundedness need not imply closedness.

Solution: Let T : D(T ) → D(T ) ⊂ X be the identity operator on D(T ), where D(T ) is
a proper dense subspace of the normed space X. Then it is immediate that T is linear
and bounded. However, we have that T is not closed. To see this, take x ∈ X \ D(T )
and a sequence {xn } ∈ D(T ) that converges to x, and then use the simple test for an
operator to be closed that we gave in class.
8. Show that the null space of a closed linear operator T : X → Y is a closed subspace of X.

Solution: Recall that the null space of a linear operator is given by Null(T ) := {x ∈
X : T x = 0}. Let x ∈ Null(T ) and suppose that xn ∈ Null(T ) is a sequence such that
xn → x. We need to show that x ∈ Null(T ).

If we have that T xn → y, then since each xn ∈ Null(T ) we have that y = 0. But


since T is closed, then we have that T x = y = 0 and so x ∈ Null(T ).

9. Let H be a Hilbert space and suppose that A : H → H is everywhere defined and

hAx, yiH = hx, AyiH ∀x, y ∈ H.

Show that A is a bounded linear operator.

Solution: It suffices to show that the graph of A is closed. So suppose that we have
(xn , Axn ) → (x, y). Then we need to show that y = Ax. To accomplish this last equality,
it is enough to show that for all z ∈ H that

hz, yiH = hz, AxiH .

But, with this observation the problem becomes rather easy:

hz, yiH = lim hz, Axn iH


n
= lim hAz, xn iH
n
= hAz, xiH
= hz, AxiH .

10. Let {Tn } be a sequence of bounded linear transformations from X → Y , where X and
Y are Banach spaces. Suppose that kTn k ≤ M < ∞ and there is a dense set E ⊂ X such
that {Tn x} converges for all x ∈ E. Show that {Tn x} converges on all of X.

Solution: Let  > 0 be given. Pick any x ∈ X. Now choose x0 ∈ E such that

kx − x0 kX < 3M , which will be possible since E is dense in X. For this x0 we have
that Tn x0 → y. So there exists an integer N such that if n, m > N then

kTn x0 − Tm x0 kY < .
3
Now, we claim that {Tn x} is Cauchy in Y , and since Y is a Banach space it is complete
and so the result follows. To see that {Tn x} is Cauchy, when n, m > N we have

kTn x − Tm xkY = kTn x − Tn x0 + Tn x0 − Tm x0 + Tm x0 − Tm xkY


≤ kTn x − Tn x0 kY + kTn x0 − Tm x0 kY + kTm x − Tm x0 kY

< kTn k kx − x0 kX + kTm k kx − x0 kX +
3
 
< 2M + = .
3M 3

You might also like